Mathcenter Forum

Mathcenter Forum (https://www.mathcenter.net/forum/index.php)
-   อสมการ (https://www.mathcenter.net/forum/forumdisplay.php?f=18)
-   -   กระทู้รวมเทคนิคการแก้โจทย์อสมการ (https://www.mathcenter.net/forum/showthread.php?t=2439)

nooonuii 13 เมษายน 2007 04:41

กระทู้รวมเทคนิคการแก้โจทย์อสมการ
 
ตามคำแนะนำของ Web Master ครับ
กระทู้นี้เป็นกระทู้รวบรวมเทคนิคการแก้โจทย์อสมการ ใครมีเทคนิคการคิดอย่างไรบ้าง ขอเชิญเข้ามาแลกเปลี่ยนความรู้กันดีกว่าครับ ผมขอเขียนหัวข้อไว้คร่าวๆก่อน แล้วจะเข้ามาเติมในตอนหลัง ใครมีเทคนิคอะไรเพิ่มเติมก็เข้ามาเติมได้เลยครับ :please:

1. การใช้อสมการพื้นฐาน
2. การใช้อสมการสำเร็จรูป/อสมการกึ่งสำเร็จรูป
3. การทำย้อนกลับ/การจัดรูปไปสู่อสมการอื่นที่สมมูลกัน
4. การแทนค่าตัวแปร
5. สมมาตรของตัวแปร
6. การใช้เอกลักษณ์พีชคณิต
ุ7. การแยกกรณี
8. การพิจารณาเงื่อนไขที่ทำให้อสมการเป็นจริง
9. การหาข้อขัดแย้ง
10. Normalization
ุ11. Homogenization
12. การแทนค่าด้วยฟังก์ชันตรีโกณมิติ
13. Calculus
14. อุปนัยเชิงคณิตศาสตร์

M@gpie 13 เมษายน 2007 23:21

ขออนุญาตเริ่มแล้วกันนะครับ ของ่ายๆก่อน อิอิ เริ่มต้นการแก้ปัญหาอสมการไม่มีหลักการตายตัวครับ ก่อนอื่นเราควรจะขีดๆเขียนๆ อสมการที่โจทย์ต้องการแล้วทำย้อนกลับไปก่อนซึ่งเป็น
เทคนิค การทำย้อนกลับ/การจัดรูปไปสู่อสมการอื่นที่สมมูลกัน ซึ่งต้องอาศัยประสบการณ์ครับ เช่น
ตัวอย่าง : ให้ $a,b,c \in \mathbb{R}$ จงพิสูจน์ว่า
\[ a^2+b^2+c^2 \geq ab+ac+bc\]
แนวคิด : ลองย้ายข้างตัวอสมการที่โจทย์ให้มา เป็น \[a^2+b^2+c^2 - ab - ac - bc \geq 0\]
เราสามารถจัดให้เป็นกำลังสองสมบูรณ์ได้ (อย่างไร?) แล้วจัดกลุ่มได้เป็น
\[ (a-b)^2+(b-c)^2+(a-c)^2 \geq 0\]
ซึ่งเป็นจริงเสมอ ถึงตอนนี้พอเริ่มพิสูจน์เราก็ทำย้อนกลับ ดังนี้
พิสูจน์ : เนื่องจาก \[ (a-b)^2+(b-c)^2+(a-c)^2 \geq 0,\;\; \forall a,b,c\in \mathbb{R}\]
กระจายกำลังสองสมบูรณ์จะได้ \[2(a^2+b^2+c^2)-2(ab+ac+bc) \geq 0 \]
จัดรูปก็จะได้อสมการที่ต้องการ
ยังมีอีกหลายตัวอย่างนะครับ อันนี้ผมทิ้งไว้เป็นโจทย์ง่ายๆนะครับให้คนที่เข้ามาอ่านลองคิดดู โดยใช้เทคนิคการทำย้อนกลับ

จงพิสูจน์ว่า
1. ให้ $a_1,a_2,x \in \mathbb{R}$ โดยที่ $0<a_1<x<a_2$ จงแสดงว่า \[\frac{1}{x}+\frac{1}{a_1+a_2-x} < \frac{1}{a_1}+\frac{1}{a_2}\]
2. ถ้า $a\geq b$ และ $x\geq y$ จงแสดงว่า $ax+by\geq ay+bx$
3. กำหนดให้ $0<x<1$ และ $0<y<1$ จงแสดงว่า $0<x+y-xy<1$

ปล. พี่ๆน้องๆท่านใดจะเฉลยก็ ซ่อนข้อความไว้ก็ดีนะครับ เผื่อว่าคนที่เข้ามาใหม่จะได้ลองคิดก่อน :great:

M@gpie 13 เมษายน 2007 23:49

หลังจากขีดๆเขียนๆแล้ว ถ้าไม่เป็นผลก็ต่อไปแนะนำลองมองหาอสมการสำเร็จรูปที่เราจำได้ มาเพื่อใช้พิสูจน์ต่อไปครับ โจทย์ที่ยากคือเราต้องใช้หลายๆทักษะประกอบกัน แล้วก็ไม่รู้ว่าจะเริ่มตรงไหนก่อน สิ่งที่ช่วยได้คือประสบการณ์ครับ (ซึ่งผมก็ไม่ค่อยจะมี)
ต่อไปก็จะแนะนำอสมการที่สำคัญและควรจดจำให้ได้ครับ จะพิสูจน์สำหรับอสมการที่ไม่ยาก ก็แล้วกันนะครับ เพราะที่พิสูจน์ยากๆผมก็ งงเหมือนกัน :d
อสมการค่าเฉลี่ยเลขคณิต-เรขาคณิต-ฮาร์โมนิก (AM $\geq$ GM $\geq$ HM) สำหรับสองจำนวน
ให้ $a,b \geq 0$ จะได้ว่า \[\frac{a+b}{2} \geq \sqrt{ab} \geq \frac{2}{\frac{1}{a}+\frac{1}{b}}\]
ข้อสังเกต สำหรับ AM $\geq$ GMอสมการกลายเป็นสมการ เมื่อ $a=b$ (ลองทำดู)
อสมการ AM $\geq$ GM $\geq $HM สามารถขยาย ไปเป็น $n$ จำนวนได้ ซึ่งการพิสูจน์สามารถทำได้หลายวิธีแต่จะยุ่งยากขอละไว้นะครับ
อสมการค่าเฉลี่ยนเลขคณิต-เรขาคณิต-ฮาร์โมนิก (AM $\geq$ GM $\geq$ HM) สำหรับ $n$ จำนวน
ให้ $a_1,a_2, ..., a_n \geq 0$ จะได้ว่า \[\frac{a_1+a_2+...+a_n}{n} \geq \sqrt{a_1a_2...a_n} \geq \frac{n}{\frac{1}{a_1}+\frac{1}{a_2}+...+\frac{1}{a_n}}\]

ตัวอย่าง : จงหาค่าต่ำสุดของฟังก์ชัน $f(x)=x+\frac{1}{x},\; \; x\in \mathbb{R}^+ $ (โดยไม่ใช้ Calculus)

ตัวอย่าง : จงหาค่าสูงสุดของฟังก์ชัน $f(x)=x(6-x)^2, \; \; x\in (0,6)$ (โดยไม่ใช้ Calculus)

M@gpie 14 เมษายน 2007 01:13

อสมการโคชี-ชวาร์ช (Cauchy-Schwarz inequality)
$x_1,x_2,...,x_n, y_1,y_2,...,y_n \in \mathbb{R}$ จะได้ว่า \[\mid x_1y_1+x_2y_2+...+x_ny_n \mid \leq \sqrt{x_1^2+x_2^2+...+x_n^2}\cdot \sqrt{y_1^2+y_2^2+...+y_n^2} \]
ความหมายเชิงเรขาคณิต ของอสมการนี้ คือ ถ้าให้ เวกเตอร์ $x = (x_1,x_2,...,x_n), y=(y_1,y_2,...,y_n)$ จะได้ว่า ผลคูณแบบดอทของสองเวกเตอร์จะมีค่าน้อยกว่าหรือเท่ากับผลคูณของขนาดของเวกเตอร์ทั้งสอง \[ \mid x\cdot y \mid \leq \|x\| \|y\|\]
การพิสูจน์อสมการนี้ผมขอแสดงวิธีที่น่าสนใจ (ที่เคยเห็นมา) คือใช้ Discriminant ของฟังก์ชัน ดังนี้
นิยาม \[ F(t)=(x_1+ty_1)^2+(x_2+ty_2)^2 +...+(x_n+ty_n)^2\]
ซึ่งจะเห็นว่า $F(t)\geq 0$ ทุกค่า $t\in \mathbb{R}$ ต่อไปทำการกระจายกำลังสองสมบูรณ์ จะได้
\[ F(t)= A+2Bt+Ct^2\]
โดยที่ $A=x_1^2+x_2^2+...+x_n^2,\;\; B=x_1y_1+x_2y_2+...+x_ny_n,\;\; C=y_1^2+y_2^2+...+y_n^2$
ซึ่งเงื่อนไข $F(t)\geq 0$ เป็นจริงได้เมื่อ $(2B)^2-4AC \leq 0$ ซึ่งทำให้ได้อสมการที่ต้องการ

ตัวอย่าง : จงแสดงว่า $\frac{a_1+a_2+...+a_n}{n} \leq \sqrt{\frac{a_1^2+a_2^2+...+a_n^2}{n}}$
(เทอมขวามือเราเรียกว่า Root Mean Square ย่อว่า RMS ครับสำหรับคนเรียนฟิสิกส์น่าจะเคยได้ยิน)

ตัวอย่าง : จงหาค่าสูงสุดของ $a\cos \theta +b\sin \theta,\;\; \theta \in \mathbb{R}$

ตัวอย่าง : จงแสดงว่า $ab+bc+ca \leq a^2+b^2+c^2$ โดยใช้อสมการ Cauchy-Schwarz
แบบฝึกหัด :
ให้ $x_1,x_2,x_3$ เป็นจำนวนจริง จงแสดงว่า
\[(\frac{1}{2}x_1+\frac{1}{3}x_2+\frac{1}{6}x_3)^2 \leq \frac{1}{2}x_1^2+\frac{1}{3}x_2^2+\frac{1}{6}x_3^2\]
ให้ $x_1,x_2,...,x_n$ เป็นจำนวนจริงบวก และ $y_1,y_2,...,y_n$ เป็นการเรียงสับเปลี่ยนชุดหนึ่งของ $x_1,x_2,...,x_n$
จงแสดงว่า \[ \frac{x_1^2}{y_1}+\frac{x_2^2}{y_2}+...+\frac{x_n^2}{y_n} \geq x_1+x_2+...+x_n\]
จงหาค่าต่ำสุดและค่าสูงสุดของ $2x+3y-6z$ เมื่อ $x,y,z\in \mathbb{R}$ ที่สองคล้องกับสมการ $x^2+y^2+z^2=1$

จากอสมการของโคชี จะทำให้เราได้ อสมการสามเหลี่ยม ดังนี้
ให้ $x_i,y_i \in \mathbb{R}$ สำหรับทุก $i = 1,2,...,n$ จะได้ว่า
\[ \sqrt{\sum_{k=1}^{n}(x_i+y_i)^2} \leq \sqrt{\sum_{k=1}^{n}x_i^2}+\sqrt{\sum_{k=1}^{n}y_i^2}\]

nooonuii 14 เมษายน 2007 11:58

ขอยกอสมการสำเร็จรูปให้น้อง Magpie ไปก็แล้วกันครับ เพราะสรุปได้เยี่ยมมากเลยครับ :great: ในเมื่อพูดถึงอสมการสำเร็จรูปแล้ว ผมขอต่อด้วยอสมการกึ่งสำเร็จรูปก็แล้วกันครับ ตั้งชื่อให้ดูเก๋ไปงั้นแหละครับ:laugh: จริงๆแล้วมันก็คืออสมการที่ได้มาจากอสมการสำเร็จรูปอีกทีนึงนั่นเอง แต่เรานำไปใช้ประโยชน์ต่อได้ด้วย ส่วนใหญ่สามารถพิสูจน์ได้โดยง่ายจากอสมการสำเร็จรูปครับ เพื่อให้ง่ายต่อการจดจำนำไปใช้จะขอเขียนเฉพาะอสมการสำหรับสามตัวแปรเท่านั้น(โจทย์ส่วนใหญ่จะเน้นอสมการสามตัวแปรครับ) ซึ่งบางอสมการอาจจะจริงสำหรับตัวแปรที่มีมากกว่าสามด้วย เอาล่ะลองดูครับว่าเรามีอสมการเหล่านี้ไว้เป็นอาวุธสำหรับทำโจทย์อสมการแล้วหรือยัง

ถ้า $a,b,c,x,y,z>0$ แล้ว

$1. \quad 3(ab+bc+ca)\leq (a+b+c)^2 \leq 3(a^2+b^2+c^2)$

$2. \quad (a+b-c)(a-b+c)(-a+b+c)\leq abc$

$3. \quad \displaystyle{ (a+b+c)\Big(\frac{1}{a}+\frac{1}{b}+\frac{1}{c}\Big)\geq 9 }$


$4. \quad \displaystyle{ \frac{a^2}{x}+\frac{b^2}{y}+\frac{c^2}{z}\geq \frac{(a+b+c)^2}{x+y+z} }$

$5. \quad 3abc(a+b+c)\leq (ab+bc+ca)^2 \leq (a^2+b^2+c^2)^2\leq 3(a^4+b^4+c^4)$

$6. \quad a^2b+b^2c+c^2a \leq a^3+b^3+c^3$

$7. \quad ab^2+bc^2+ca^2 \leq a^3+b^3+c^3$

$8. \quad ab(a+b)\leq a^3+b^3$

เอาไว้คิดอะไรเพิ่มได้อีกจะมาต่อให้ครับ

M@gpie 15 เมษายน 2007 15:11

ก่อนอื่นพิจารณา Lemma ที่จะนำไปพิสูจน์อสมการอื่นต่อไป และ เป็นตัวอย่างที่ใช้แคลคูลัสในการพิสูจน์ ดังนี้
Lemma : ให้ $t \in (0,1)$ และ $x,y \geq 0$ จะได้ว่า
\[ x^t y^{1-t} \leq tx +(1-t)y\]
จะใช้เทคนิคทางแคลคูลัส ในการพิสูจน์อสมการนี้ โดยพิจารณาดังนี้
กรณีที่ $x=0$ หรือ $y=0$ จะได้ว่าอสมการเป็นจริง ดังนั้นสมมติให้ $x>0,y>0$ เราจะพิสูจน์อสมการที่สมมูลกันแทนดังนี้ (เอา $y$ หารตลอด)
\[ (\frac{x}{y})^t \leq t(\frac{x}{y}) +1-t\]
สมมติให้ $f(x) = x^t-tx+t-1, \;\; x > 0, t\in (0,1)$ จะได้ว่า $f'(x)=tx^{t-1}-t =t(x^{t-1}-1)$
และ $f''(x)=t(t-1)x^{t-2}$ จะเห็นว่า $f'(x)=0$ ก็ต่อเมื่อ $x=1$ ซึ่งเป็นค่าวิกฤตเพียงค่าเดียว และ $f''(1) <0$ จึงได้ว่า $f$ ให้ค่าสูงสุดสัมบูรณ์ที่ $x=1$ จึงได้ว่า
\[x^t-tx+t-1 =f(x) \leq f(1) = 0 , \forall x>0\] ซึ่ง เป็นอสมการที่ต้องการโดยเปลี่ยนตัวแปร $x$ ไปเป็น $\frac{x}{y}$
จาก Lemma ข้างบนทำให้ได้ อสมการค่าเฉลี่ยเลขคณิตและเรขาคณิตถ่วงน้ำหนัก สำหรับสองจำนวน ดังนี้
สำหรับ $x_1,x_2 \geq 0$ และ $w_1,w_2 \in \mathbb{R}^+$ ซึ่ง $w_1+w_2 =1$
\[ x_1^{w_1}x_2^{w_2} \leq w_1x_1+w_2x_2\]
และใช้การอุปนัยขยายไปเป็น $n$ จำนวนดังนี้
สำหรับ $x_1,x_2,...,x_n \geq 0$ และ $w_1,w_2,...,w_n \in \mathbb{R}^+$ ซึ่ง $w_1+w_2+...+w_n =1$
\[ x_1^{w_1}x_2^{w_2}\cdot ...\cdot x_n^{w_n} \leq w_1x_1+w_2x_2+...+w_nx_n\]
และอสมการเป็นสมการเมื่อ $x_1=x_2=...=x_n$
ตัวอย่างติดไว้ก่อนครับ เดี๋ยวมาเพิ่มให้ (แอบไปอ่านก่อนแหะๆ :cool: )

[Tong]_1412 16 เมษายน 2007 00:22

ถามไรหน่อยนะคับ อสมการที่คุณ nooonuii ยกมานิ สามารถใช้ได้เลยหรือคับ บางอันก็พิสูจน์ได้ แต่บางอันก็ยังงงๆ อ่าคับ

M@gpie 16 เมษายน 2007 00:42

อสมการที่พี่ noonuii แนะนำ ก็เป็นเพียงอสมการผลลัพธ์จาก อสมการสำเร็จรูปครับ ซึ่งเอาไปใช้งานได้เลย (ถ้าจำได้) แต่ผมจำไม่ค่อยได้ครับ ลองพิสูจน์ดูเป็นแบบฝึกหัดเลยจะดีมากครับ :D

nooonuii 16 เมษายน 2007 00:45

อ้างอิง:

ข้อความเดิมเขียนโดยคุณ [Tong]_1412 (ข้อความที่ 17850)
ถามไรหน่อยนะคับ อสมการที่คุณ nooonuii ยกมานิ สามารถใช้ได้เลยหรือคับ บางอันก็พิสูจน์ได้ แต่บางอันก็ยังงงๆ อ่าคับ

ใช้ได้ครับ แต่ถ้าพิสูจน์ได้ด้วยก็จะดีมากครับ เดี๋ยวผมแทรกบทพิสูจน์ไว้ให้ดูด้วยครับ :)

[Tong]_1412 17 เมษายน 2007 00:54

ขอบคุณมากนะคับ เด่วผมดูแนวคิดเฉยๆ ดีกว่า จำไม่ได้อ่าคับ ^^]

ขอถามไรอีกอย่างได้ไหมคับ "เป็นผลโดยตรงจากอสมการการจัดเรียง"
อสมการการจัดเรียง คืออะไรอ่าคับ
แต่หลังจากอ่านของคุณ nooonuii ทำให้เข้าใจมากขึ้นไงก็ขอบคุณมากนะคับ ^^

nooonuii 19 เมษายน 2007 07:38

อ้างอิง:

ข้อความเดิมเขียนโดยคุณ [Tong]_1412 (ข้อความที่ 17894)
ขอถามไรอีกอย่างได้ไหมคับ "เป็นผลโดยตรงจากอสมการการจัดเรียง"
อสมการการจัดเรียง คืออะไรอ่าคับ
^^

อสมการการจัดเรียง มีอยู่ในหนังสืออสมการของสอวน.เรียบร้อยแล้วครับ ลองศึกษาดูก่อนครับ:)

nooonuii 19 เมษายน 2007 08:22

5. สมมาตรของตัวแปร

โดยไม่เสียนัยทั่วไป (Without loss of generality หรือเขียนสั้นๆว่า WLOG) เป็นคำพูดสั้นๆที่ใช้แทนการเขียนพิสูจน์โดยการเลือกพิสูจน์กรณีเฉพาะกรณีหนึ่ง แต่บทพิสูจน์สามารถเป็นตัวแทนสำหรับการพิสูจน์ในกรณีอื่นๆและครอบคลุมกรณีที่เป็นไปได้ทั้งหมด

สำหรับการพิสูจน์อสมการนั้น เราไม่สามารถใช้คำนี้ได้เสมอไ้ป แต่จะมีอสมการอยู่ประเภทหนึ่งซึ่งเราสามารถพิสูจน์เพียงกรณีเดียวแต่ส่งผลให้อสมการเป็นจริงในกรณีที่เหลือทั้งหมด อสมการประเภทนี้เราเรียกว่า อสมการที่มีสมมาตรในตัวแปร โดยทั่วไปการพิสูจน์อสมการนั้นเราอาจจะต้องเรียงค่าให้กับตัวแปรเพื่อนำเอาข้อสมมติจากการเรียงค่าให้ตัวแปรมาใช้ประโยชน์ด้วย แต่การแยกกรณีจะทำให้เราต้องเขียนบทพิสูจน์เยอะมากซึ่งไม่สะดวก สำหรับอสมการที่มีสมมาตรในตัวแปรเราสามารถเลือกพิสูจน์เพียงกรณีเดียว และโดยไม่เสียนัยทั่วไป อสมการจะจริงสำหรับทุกกรณี คราวนี้อสมการแบบไหนที่เราเรียกว่ามีสมมาตรในตัวแปร ?

อสมการที่มีสมมาตรในตัวแปร คือ อสมการซึ่งไม่ว่าเราจะเรียงสับเปลี่ยนตัวแปรด้วยวิธีใดก็ตามเราก็ยังคงได้อสมการเดิม เช่น อสมการ

$$\frac{a}{b+c}+\frac{b}{c+a}+\frac{c}{a+b}\geq\frac{3}{2}$$

ถ้าเราเรียงสับเปลี่ยนตัวแปรด้งนี้ $a\to c, b\to a, c\to b$ แล้วแทนค่ากลับไปเราจะได้อสมการ

$$\frac{c}{a+b}+\frac{a}{b+c}+\frac{b}{c+a}\geq\frac{3}{2}$$

ซึ่งจะเห็นว่ายังคงเป็นอสมการเดิม สำหรับการเรียงสับเปลี่ยนตัวแปรวิธีอื่นๆ(วิธีเรียงสับเปลี่ยนตัวแปรสำหรับสามตัวแปรมีอยู่ทั้งหมด 6 วิธี) ก็จะให้อสมการเดียวกัน อสมการนี้จึงมีสมมาตรในตัวแปร

ต่อไปลองดูอสมการนี้บ้าง

$$a^3+3b^3+9c^3 \geq 9abc $$

ถ้าเราทำการเรียงสับเปลี่ยนตัวแปรเสียใหม่ดังนี้ $a\to b,b\to c,c\to a$ เราจะได้อสมการเป็น $$b^3+3c^3+9a^3 \geq 9bca$$

ซึ่งจะเห็นว่าเราไม่ได้อสมการเดิม อย่างนี้ถือว่าอสมการไม่มีสมมาตรในตัวแปร ครับ

nooonuii 28 เมษายน 2007 23:50

6. เอกลักษณ์พีชคณิต

เอกลักษณ์พีชคณิตมีความสำคัญอย่างยิ่งยวดในการแก้โจทย์อสมการพีชคณิตครับ มีเอกลักษณ์ที่น่าสนใจและนำมาใช้อยู่บ่อยๆดังนี้

1. $(a+b+c)^2 = (a^2+b^2+c^2) + 2(ab+bc+ca)$

2. $(a+b+c)^3 = (a^3+b^3+c^3) + 3(a^2b+b^2c+c^2a+ab^2+bc^2+ca^2) + 6abc$

3. $(a+b+c)^3 = (a^3+b^3+c^3) + 3(a+b)(b+c)(c+a)$

4. $a^3+b^3+c^3 = 3abc + (a+b+c)(a^2+b^2+c^2-ab-bc-ca)$

5. $(1+a)(1+b)(1+c) = 1 + (a+b+c) + (ab+bc+ca) + abc$

6. $(1-a)(1-b)(1-c) = 1 - (a+b+c) + (ab+bc+ca) - abc$

7. $(a+b)(b+c)(c+a) + abc = (a+b+c)(ab+bc+ca)$

8. $(a^2+b^2)(c^2+d^2) = (ad-bc)^2 + (ac+bd)^2 = (ac-bd)^2 + (ad+bc)^2$

9. $(a+b)(b+c)(c+a)+(1+a)(1+b)(1+c) = (1+a+b+c)(1+ab+bc+ca)$

10. $(a+b+c)\Big(\dfrac{1}{a}+\dfrac{1}{b}+\dfrac{1}{c}\Big)=3 + \Big(\dfrac{a}{b}+\dfrac{b}{c}+\dfrac{c}{a}\Big)+\Big(\dfrac{a}{c}+\dfrac{c}{b}+\dfrac{b}{a}\Big)$

kanakon 29 เมษายน 2007 20:17

hint แบบฝึก
1 $\because$ $a_1$ $(a_2 - x)$ < $x$ $(a_2 - x)$

2. $\because$ $x$ $(a - b)$ $\geq $ $y$ $(a - b)$

3. 0<x<1 และ 0<y<1 และ 0<xy<1

ThirdkunG 30 เมษายน 2007 17:30

ขอบคุณมากๆงับพี่ๆๆทุกคน


เวลาที่แสดงทั้งหมด เป็นเวลาที่ประเทศไทย (GMT +7) ขณะนี้เป็นเวลา 17:13

Powered by vBulletin® Copyright ©2000 - 2024, Jelsoft Enterprises Ltd.
Modified by Jetsada Karnpracha